Difference between revisions of "2002 AMC 10B Problems/Problem 7"

(still needs solution)
 
Line 1: Line 1:
 
== Problem ==
 
== Problem ==
  
Let <math>n</math> be a positive integer such that <math>\frac {1}{2} + \frac {1}{3} + \frac {1}{7} + \frac {1}{n}</math> is an integer. Which of the following statements is [b]not[/b] true?
+
Let <math>n</math> be a positive integer such that <math>\frac {1}{2} + \frac {1}{3} + \frac {1}{7} + \frac {1}{n}</math> is an integer. Which of the following statements is ''not'' true?
  
 
<math> \mathrm{(A) \ } 2\text{ divides }n\qquad \mathrm{(B) \ } 3\text{ divides }n\qquad \mathrm{(C) \ } 6\text{ divides }n\qquad \mathrm{(D) \ } 7\text{ divides }n\qquad \mathrm{(E) \ } n>84 </math>
 
<math> \mathrm{(A) \ } 2\text{ divides }n\qquad \mathrm{(B) \ } 3\text{ divides }n\qquad \mathrm{(C) \ } 6\text{ divides }n\qquad \mathrm{(D) \ } 7\text{ divides }n\qquad \mathrm{(E) \ } n>84 </math>

Revision as of 02:47, 27 December 2008

Problem

Let $n$ be a positive integer such that $\frac {1}{2} + \frac {1}{3} + \frac {1}{7} + \frac {1}{n}$ is an integer. Which of the following statements is not true?

$\mathrm{(A) \ } 2\text{ divides }n\qquad \mathrm{(B) \ } 3\text{ divides }n\qquad \mathrm{(C) \ } 6\text{ divides }n\qquad \mathrm{(D) \ } 7\text{ divides }n\qquad \mathrm{(E) \ } n>84$